Đến nội dung

Khoa Linh nội dung

Có 599 mục bởi Khoa Linh (Tìm giới hạn từ 12-06-2020)



Sắp theo                Sắp xếp  

#711213 [TOPIC] HÌNH HỌC ÔN THI VÀO THPT CHUYÊN 2018-2019

Đã gửi bởi Khoa Linh on 19-06-2018 - 14:19 trong Tài liệu - Đề thi

Bài 24. Cho tam giác $ABC$, $M,N,P$ là các điểm bất kì trên các cạnh $BC,CA,AB$. Chứng minh rằng trong các tam giác $ANP, BMP, CMN$ tồn tại một tam giác có diện tích không vượt quá $\frac{1}{4}$ diện tích tam giác $ABC$.

một kết quả khác khi $M,N,P$ là chân các đường phân giác.

Ta có: 
$\dfrac{S_{ANP}}{S_{ABC}}=\dfrac{AP.AN}{AB.AC}$
 
$\dfrac{S_{CMN}}{S_{ABC}}=\dfrac{CN.CM}{CA.CB}$; $\dfrac{S_{BMP}}{S_{ABC}}=\dfrac{BP.BM}{BA.BC}$
 
Suy ra: $\dfrac{S_{ANP}.S_{CMN}S_{BMP}}{(S_{ABC})^3}=\dfrac{AP.BP}{AB^2}.\dfrac{CN.AN}{AC^2}.\dfrac{CM.BM}{BC^2} \leq \dfrac{1}{64}$
 
Vậy trong 3 tam giác $ANP, BMP, CMN$ tồn tại một tam giác có diện tích không vượt quá $\dfrac{1}{4}$ diện tích tam giác $ABC$.

Hình gửi kèm

  • bai24.jpg



#710836 Cho A=(a+b)^{2}-2a^{2};B=(a+b)^{2}-2b^{2...

Đã gửi bởi Khoa Linh on 14-06-2018 - 00:28 trong Số học

Với a,b là các số nguyên dương $Cho A=(a+b)^{2}-2a^{2};B=(a+b)^{2}-2b^{2}.Cm A,B không đồng thời là số chính phương$

Bài này thi chuyên Sư Phạm bạn nhé.

Lời giải ở đây: 

File gửi kèm




#710829 $\sqrt{x^{2}+y^{2}}+\frac{1...

Đã gửi bởi Khoa Linh on 13-06-2018 - 21:58 trong Bất đẳng thức và cực trị

 




#710824 $\sum \frac{1}{a^{2}+abc}$

Đã gửi bởi Khoa Linh on 13-06-2018 - 21:49 trong Bất đẳng thức và cực trị

Cho $a,b,c\geq 0;a+b+c=3$. Tìm $Min$ của $\sum \frac{1}{a^{2}+abc}$

Áp dụng $AM-GM$ ta có:

$\frac{1}{a(a+bc)}+\frac{bc+a}{4}\geq \frac{1}{\sqrt{a}}$

Suy ra: 

$\sum \frac{1}{a^2+abc}\geq \sum \frac{1}{\sqrt{a}}-\frac{ab+bc+ca+a+b+c}{4}$

Ta lại có: 

$\sum \frac{1}{\sqrt{a}}\geq \frac{9}{\sqrt{a}+\sqrt{b}+\sqrt{c}}$

Vì $a+b+c=3$ nên $\sqrt{a}+\sqrt{b}+\sqrt{c}\leq 3;ab+bc+ca\leq 3$

Suy ra: $\sum \frac{1}{\sqrt{a}}-\frac{ab+bc+ca+a+b+c}{4}\geq \frac{3}{2}$

p/s: Em dùng lắm BĐT như này mà không bị ngược dấu nhỉ ??. Thấy sai sai, anh kiểm tra lại nhé 




#710719 $f(x)=(x^4+\sqrt{2}x-7)^{2018}$

Đã gửi bởi Khoa Linh on 12-06-2018 - 21:33 trong Đại số

Cho hàm số $f(x)=(x^4+\sqrt{2}x-7)^{2018}$. Tính $f(a)$ với $a=(4+\sqrt{15})(\sqrt{5}-3)\sqrt{4-\sqrt{15}}$

( Hơi lẻ ạ )

Chỗ bôi đỏ phải là $\sqrt{5}-\sqrt{3}$

Ta có: 

$(4+\sqrt{15})(\sqrt{5}-\sqrt{3}).\sqrt{4-\sqrt{15}}=\sqrt{4+\sqrt{15}}.(\sqrt{5}-\sqrt{3}).\sqrt{(4-\sqrt{15})(4+\sqrt{15})}=\sqrt{4+\sqrt{15}}.(\sqrt{5}-\sqrt{3})=\sqrt{\frac{(\sqrt{5}+\sqrt{3})^2}{2}}.(\sqrt{5}-\sqrt{3})=\sqrt{2}$

Suy ra $f(x)=(x^4+\sqrt{2}x-7)^{2018}=1$




#710717 $\sqrt{a+b}=\sqrt{a+c}+\sqrt{b+c...

Đã gửi bởi Khoa Linh on 12-06-2018 - 21:25 trong Đại số

Cho 2 số dương $a,b$. Số $c$ là một số khác 0 thỏa mãn $\frac{1}{a}+\frac{1}{b}+\frac{1}{c}=0$. CMR

$\sqrt{a+b}=\sqrt{a+c}+\sqrt{b+c}$

Trước hết ta lập luận: Vì $a,b>0$ nên $c<0$

Suy ra $\frac{1}{b}+\frac{1}{c}<0\Leftrightarrow \frac{b+c}{bc}<0\Rightarrow b+c>0$

Tương tự nên ta có: $c+a>0$

 

Từ giả thiết suy ra $ab+bc+ca=0$

$\Rightarrow (c+a)(c+b)=c^2\Rightarrow 2\sqrt{(c+a)(c+b)}=-2c$ (vì $c<0$)

$\Leftrightarrow (c+a)+(c+b)+2\sqrt{(c+a)(c+b)}=a+b\Rightarrow \left ( \sqrt{c+a}+\sqrt{c+b} \right )^2=a+b\Rightarrow$ đpcm




#710716 $x=\sqrt{2+\sqrt{3}}-\sqrt{...

Đã gửi bởi Khoa Linh on 12-06-2018 - 21:19 trong Đại số

Cho $x=\sqrt{2+\sqrt{3}}-\sqrt{\frac{\sqrt[3]{6\sqrt{3}-10}}{\sqrt{3}+1}}$. Tính $A=(x^4+x^3-x^2-2x-1)^{2019}$

Ta có:

 

$\sqrt{2+\sqrt{3}}-\sqrt{\frac{\sqrt[3]{6\sqrt{3}-10}}{\sqrt{3}+1}}=\frac{\sqrt{4+2\sqrt{3}}}{\sqrt{2}}-\sqrt{\frac{\sqrt[3]{(\sqrt{3}-1)^3}}{\sqrt{3}+1}}=\frac{\sqrt{3}+1}{\sqrt{2}}-\sqrt{\frac{\sqrt{3}-1}{\sqrt{3}+1}}$

$=\frac{1+\sqrt{3}}{\sqrt{2}}-\frac{\sqrt{3}-1}{\sqrt{2}}=\frac{2}{\sqrt{2}}=\sqrt{2}$

Suy ra $x=\sqrt{2}\Rightarrow x^4+x^3-x^2-2x-1=1\Rightarrow A=1$




#710707 Đề thi vào lớp 10 THPT tỉnh Hà Nam (toán chung)

Đã gửi bởi Khoa Linh on 12-06-2018 - 20:21 trong Tài liệu - Đề thi

Vì $a,b,c$ là độ dài 3 cạnh tam giác có chu vi là 4 nên $0<a,b,c<2$.

Ta đi chứng minh:  $\frac{1}{a}+2a>\frac{9}{4-a}$ $\Leftrightarrow (a-1)^2(a-2)<0$ (đúng)

Thiết lập các BĐT tương tự ta có đpcm 




#710670 $x^{2}+y^{3}\geq x^{3}+y^{4...

Đã gửi bởi Khoa Linh on 11-06-2018 - 23:55 trong Bất đẳng thức và cực trị

Cho $x,y$ là các số thực dương thỏa mãn $x^{2}+y^{3}\geq x^{3}+y^{4}$. Chứng minh rằng $x^{3}+y^{3}\leq 2$

Áp dụng $AM-GM$ ta có: 

$x^3+x^3+1\geq 3x^2$

$y^4+y^4+y^4+1\geq 4y^3$

Suy ra $2x^3+3y^4+2\geq y^3+3(x^2+y^3)\geq y^3+3(x^3+y^4)\Rightarrow x^3+y^3\leq 2$




#710500 [TOPIC] HÌNH HỌC ÔN THI VÀO THPT CHUYÊN 2018-2019

Đã gửi bởi Khoa Linh on 10-06-2018 - 20:56 trong Tài liệu - Đề thi

Hôm nay là tròn 1 tháng từ khi TOPIC hình học được thành lập. Chúng ta có vẻ là đã trải qua hầu hết các kì thi THPT trên các tỉnh toàn cả nước. TOPIC cũng đã gặt hái nhiều thành công nhất định, chúng ta đã trải qua rất nhiều bài toán hay khác nhau cùng với nhiều lời giải đẹp. Mình xin chân thành cảm ơn sự đóng góp của các thành viên đã giúp cho TOPIC phát triển. Tuy vậy vẫn còn tồn tại khá nhiều bài toán hình hay chưa có lời giải, mình mong các bạn xem lại các bài toán mà các bạn đưa lên, nếu có lời giải thì cần post lên để giúp TOPIC hoàn thiện hơn. Cuối cùng, mình xin tuyên bố kết thúc TOPIC và sẽ không đăng bài lên nữa. Chúc các bạn có một mùa hè vui vẻ, bổ ích, các cuộc thi đã qua rồi hãy gác lại phía sau nhé. Thân mến!

Khoa Linh -




#710408 Chứng minh $KC=KD$

Đã gửi bởi Khoa Linh on 09-06-2018 - 23:04 trong Hình học

Vì sao có $CEFD$ là hình thang, $K$ là trung điểm $EF$ thì suy ra được đpcm vậy bạn?

Chịu khó nghĩ tí đi bạn, đáng ra bài này nhìn hình là hiểu ý tưởng rồi...

Hình gửi kèm

  • Untitled.png



#710405 Chứng minh $KC=KD$

Đã gửi bởi Khoa Linh on 09-06-2018 - 22:56 trong Hình học

Cho 2 đường tròn $(O)$ và $(O')$ cắt nhau tại $A$ và $B$. Gọi $K$ là điểm sao cho $OAO'K$ là hình bình hành. Điểm $C$ thuộc $(O)$. $CA$ cắt $(O')$ tại $D$. Chứng minh $KC=KD$

Bài này khá quen.

Ta có: $OO'$ đi qua trung điểm $AB,AK$ từ đó suy ra $OO' $ || $BK$

$BK$ cắt hai đường tròn tại $E,F$.

Khi đó $AE,AF$ là đường kính và ta có: $CEFD$ là hình thang. 

Kết hợp $K$ là trung điểm $EF$ (dễ chứng minh). Từ đó suy ra đpcm

 

Một mở rộng nho nhỏ: Đường tròn $(K;KD)$ cắt $(O),(O')$ tại $M,N$ thì $M,A,N$ thẳng hàng

Hình gửi kèm

  • Untitled.png



#710393 $3\sqrt{3}(x^2+4x+2)-\sqrt{x+8}=0$

Đã gửi bởi Khoa Linh on 09-06-2018 - 21:06 trong Phương trình, hệ phương trình và bất phương trình

Câu 1: Giải PT 

$3\sqrt{3}(x^2+4x+2)-\sqrt{x+8}=0$

(trích đề thi tuyển sinh THái Bình 2018-2019)

Bình phương rồi dùng Casio tìm nhân tử

 34686250_952881684893417_417420702338829




#710392 Cho các số thực dương a,b,c. Chứng minh: $\sum \frac{a^2+...

Đã gửi bởi Khoa Linh on 09-06-2018 - 21:03 trong Bất đẳng thức và cực trị

34693652_630213893979471_516793889376855




#710301 Đề + Đáp án môn toán THPT chuyên Hùng Vương tỉnh Phú Thọ 2018-2019

Đã gửi bởi Khoa Linh on 08-06-2018 - 16:15 trong Tài liệu - Đề thi

https://khoalinhmath...huyen-hung.html

Hình gửi kèm

  • 34684265_597958950578107_2044325113439977472_n.jpg

File gửi kèm




#710255 cm AH = 3HD

Đã gửi bởi Khoa Linh on 07-06-2018 - 22:46 trong Hình học

Ta có: $\triangle DEH \sim \triangle BEA(g.g)\Rightarrow ED=\sqrt{5}EH$

Đặt $EH=x$. Áp dụng hệ thức lượng ta có: $ED^2=EH.EC\Rightarrow EC=5x\Rightarrow AE=5x\Rightarrow AH=6x$

Từ đó suy ra $AH=3HD$

Hình gửi kèm

  • Untitled.png



#710151 $\sum \frac{\sqrt{ab}}{c+2\...

Đã gửi bởi Khoa Linh on 06-06-2018 - 22:08 trong Bất đẳng thức và cực trị

Cho mình hỏi bài này luôn: Cho $0<x<1$ và $0<y<1$ và $\frac{x}{1-x}+\frac{y}{1-y}=1$. Tính $P=x+y+\sqrt{x^2-xy+y^2}$

Quy đồng giả thiết ta sẽ có: 

$-3xy=1-2(x+y)$

Suy ra $P=x+y+\sqrt{x^2-xy+y^2}=x+y+\sqrt{(x+y)^2-3xy}=x+y+\sqrt{(x+y)^2-2(x+y)+1}=x+y+\left | 1-(x+y) \right |$

Mặt khác: 

$\frac{x}{1-x}+\frac{y}{1-y}=1\Rightarrow \frac{x}{1-x}<1\Rightarrow 2x<1$. Tương tự $2y<1$ rồi suy ra $x+y<1$ 

Vậy từ đó ta có: $P=1$.

p/s: Lần sau tạo chủ đề mới nhé 




#710150 $\frac{bc}{a^2+1}+\frac{ac}...

Đã gửi bởi Khoa Linh on 06-06-2018 - 21:56 trong Bất đẳng thức và cực trị

Cho $a,b,c$ là các số thực dương thỏa mãn $a^2+b^2+c^2=1$. CMR

$\frac{bc}{a^2+1}+\frac{ac}{b^2+1}+\frac{ba}{c^2+1}\leq \frac{3}{4}$

Tác giả: Phạm Kim Hùng

Ta có: 

$\sum \frac{bc}{a^2+1}\leq \frac{1}{4}\sum \frac{(b+c)^2}{a^2+(b^2+c^2+a^2)}\leq \frac{1}{8}\sum \left ( \frac{b^2}{a^2+b^2}+\frac{c^2}{c^2+a^2} \right )=\frac{3}{4}$




#710121 $P=\sqrt{\frac{a}{a+1}}+\sq...

Đã gửi bởi Khoa Linh on 06-06-2018 - 19:15 trong Bất đẳng thức và cực trị

Cho các số không âm $a,b,c$ thỏa mãn $a+b+c=1$. 

Tìm GTLN, GTNN của biểu thức: 

$P=\sqrt{\frac{a}{a+1}}+\sqrt{\frac{b}{b+1}}+\sqrt{\frac{c}{c+1}}$

(Sưu tầm)




#710103 $\frac{ab}{a+b-c}+\frac{bc}...

Đã gửi bởi Khoa Linh on 06-06-2018 - 13:10 trong Bất đẳng thức và cực trị

Cho $a,b,c$ là độ dài 3 cạnh của một tam giác có chu vi là 3. Chứng minh rằng: 

$\frac{ab}{a+b-c}+\frac{bc}{b+c-a}+\frac{ca}{c+a-b}\geq 3$

-Sáng tác-

Ta viết lại BĐT dưới dạng: 

$\sum \frac{4ab}{a+b-c}\geq 4(a+b+c)\Leftrightarrow \sum \left [ \frac{4ab}{a+b-c}-(a+b+c) \right ]\geq a+b+c$

$\sum \frac{(a+c-b)(b+c-a)}{(a+b-c)}\geq a+b+c$

Bất đẳng thức trên là BĐT quen thuộc sau: $\frac{xy}{z}+\frac{yz}{x}+\frac{zx}{y}\geq x+y+z$




#710067 Đề thi tuyển sinh vào lớp 10 chuyên ĐHSP Hà Nội năm 2018-2019

Đã gửi bởi Khoa Linh on 05-06-2018 - 21:33 trong Tài liệu - Đề thi

Lời giải tại đây:

File gửi kèm




#710053 $\frac{ab}{a+b-c}+\frac{bc}...

Đã gửi bởi Khoa Linh on 05-06-2018 - 20:17 trong Bất đẳng thức và cực trị

Cho $a,b,c$ là độ dài 3 cạnh của một tam giác có chu vi là 3. Chứng minh rằng: 

$\frac{ab}{a+b-c}+\frac{bc}{b+c-a}+\frac{ca}{c+a-b}\geq 3$

-Sáng tác-




#710047 [TOPIC] ÔN THI BẤT ĐẲNG THỨC $\boxed{\text{THPT CHUYÊN}}$...

Đã gửi bởi Khoa Linh on 05-06-2018 - 19:40 trong Tài liệu - Đề thi

Bên đó thi chuyên chưa?

$\boxed{\text{Bài 143}}$ Cho $a,b,c >0$ ;abc=1.

Chứng minh $\frac{1}{(1+a)^2}+\frac{1}{(1+b)^2}+\frac{1}{(1+c)^2}+\frac{2}{(1+a)(1+b)(1+c)} \geq 1$

 

 

Lời giải của mình khá "vất vả" nhưng đem lại hiệu quả: 

Quy đồng ta có BĐT tương đương:

$\sum (a+1)^2(b+1)^2+2(a+1)(b+1)(c+1)\geq (a+1)^2(b+1)^2(c+1)^2$

$\Leftrightarrow (a+b+c)^2+4(a+b+c)-6abc-a^2b^2c^2-2abc(ab+bc+ca)-4abc(a+b+c)+4 \geq 0$

Thay $abc=1$ ta có BĐT trở thành: $a^2+b^2+c^2\geq 3$ (đúng theo AM-GM)

p/s: Bên mình sắp thi rồi, mùng 7,8 tháng 6. Mong bạn sớm đưa ra lời giải hay hơn  :D  :D




#710046 chứng minh (a+1)(b+1)(c+1)<=27

Đã gửi bởi Khoa Linh on 05-06-2018 - 19:34 trong Bất đẳng thức và cực trị

Cho a , b , c > 0, biết rằng a2 + b2 + c2 = 12

 

Chứng minh: ( a + 1 )( b + 1 )( c + 1 ) $\leq$ 27

Ta có:

$(a+b+c)^2\leq 3(a^2+b^2+c^2)=36\Rightarrow a+b+c\leq 6$

Áp dụng AM-GM ta có: 

$(a+1)(b+1)(c+1)\leq \frac{(a+b+c+3)^3}{27}\leq \frac{9^3}{27}=27$




#710045 AB+AB=3BC

Đã gửi bởi Khoa Linh on 05-06-2018 - 19:32 trong Hình học

có phương pháp nào để kẻ đường phụ ko anh? hay làm nhiều mới có! Em sắp thi rồi mà kém nhất là phần hình. : :botay

Thực sự hình học là một mảng trừu tượng, có nhiều lời giải khá mất tự nhiên. 

Về việc kẻ thêm thì phải làm nhiều, dựa vào kinh nhiệm đã có thôi và một chút dự đoán, may mắn nữa ...
Chúc bạn thành công